subject
Mathematics, 03.06.2020 13:18 Richelleypinelley

WILL GIVE BRAINLIEST TO BEST ANSWER Use the figure to answer the question that follows:
Segments UV and WZ are parallel with line ST intersecting both at points Q and R respectively
When written in the correct order, the two-column proof below describes the statements and reasons for proving that corresponding angles are congruent:
StatementsReasons
segment UV is parallel to segment WZGiven
Points S, Q, R, and T all lie on the same line. Given
Im∠SQT = 180°Definition of a Straight Angle
IIm∠SQV + m∠VQT = 180°Substitution Property of Equality
IIIm∠SQV + m∠VQT = m∠SQTAngle Addition Postulate
m∠VQT + m∠ZRS = 180°Same-Side Interior Angles Theorem
m∠SQV + m∠VQT = m∠VQT + m∠ZRSSubstitution Property of Equality
m∠SQV + m∠VQT − m∠VQT = m∠VQT + m∠ZRS − m∠VQT
m∠SQV = m∠ZRSSubtraction Property of Equality
∠SQV ≅ ∠ZRSDefinition of Congruency
Which is the most logical order of statements and reasons I, II, and III to complete the proof? (5 points)

Select one:
a. I, III, II
b. II, I, III
c. II, III, I Incorrect
d. III, I, II

ansver
Answers: 1

Another question on Mathematics

question
Mathematics, 21.06.2019 14:30
Which of these people has balanced their checkbook correctly? oa. gary: the balance in his check register is $500 and the balance in his bank statement is $500. b. gail: the balance in her check register is $400 and the balance in her bank statement is $500. c. gavin: the balance in his check register is $500 and the balance in his bank statement is $510.
Answers: 2
question
Mathematics, 21.06.2019 20:30
1) you deposit $2,500 in an account that earns 4% simple interest. how much do you earn in eight years?
Answers: 1
question
Mathematics, 21.06.2019 21:00
Hurry if the population of of an ant hill doubles every 10 days and there are currently 100 ants living in the ant hill what will the ant population be in 20 days
Answers: 2
question
Mathematics, 21.06.2019 23:30
Find the arc length of the semicircle. 7the grade
Answers: 1
You know the right answer?
WILL GIVE BRAINLIEST TO BEST ANSWER Use the figure to answer the question that follows:
Segme...
Questions
question
World Languages, 21.07.2019 11:30